Autor Tema: Convergencia de la serie geométrica.

0 Usuarios y 1 Visitante están viendo este tema.

13 Enero, 2022, 08:31 am
Leído 497 veces

Samir M.

  • Physicsguy.
  • $$\Large \color{#5b61b3}\pi\,\pi\,\pi\,\pi\,\pi$$
  • Mensajes: 1,087
  • País: es
  • Karma: +0/-0
  • Sexo: Masculino
  • I'm back.
Probar que la serie geométrica, definida como la sucesión \[  \{1+x+x^2+\cdots+x^n\} = \sum_{n\ge 0}x^n \] converge a \( \dfrac{1}{1-x} \) para \( |x|<1 \). Dar un argumento para deducir la expresión a la que converge la serie, \( \dfrac{1}{1-x} \).
_______

Para que la serie pueda ser convergente, es necesario que la sucesión \( \{x^n\} \) converga a \( 0 \). Si \[ |x|>1 \] entonces \( \{x^n\} \) no está acotada y no puede ser convergente. Si \( x=-1 \) entonces la sucesión \( \{x^n\} \) tiene dos sucesiones parciales que tienden a distinto límite. Si \[ x=1 \] entonces \[ \{x^n\} \] no tiende a \[ 0 \] sino a \[ 1 \]. Para \[ |x|<1 \] la sucesión converge a cero. En efecto: para \[ |x|<1 \] podemos escribir \( |x| \) como \( |x|=\dfrac{1}{1+\rho} \) para un \( \rho>0 \) adecuado y así \( |x|^n=\dfrac{1}{(1+\rho)^n} \). Veamos si \( \{x^n\} \) converge a cero. Usando la desigualdad de las medias con \( x_1,x_2, \cdots, x_{n-1}=1  \) y \( x_n = 1+n\rho \) tenemos que

\( \sqrt{\underbrace{1\cdot 1 \cdot 1 \cdots 1}_{n-1 \text{ veces}}\cdot(1+n\rho)} < \dfrac{(n-1)+(1+n\rho)}{n} = 1+\rho \) luego \( (1+\rho)^n > 1+n\rho \).


Así, dado un \( \epsilon > 0 \) tenemos que \( \displaystyle \left|x^{n}-0\right|=|x|^{n}=\frac{1}{(1+\rho)^{n}} \leqslant \frac{1}{1+n \rho}<\frac{1}{n \rho} \) y por tanto, si \( m=\dfrac{1}{\rho\epsilon} \) o, mejor aún, si \( m = E\left(\frac{1}{\rho \epsilon}\right)+1 \) donde \( E(x) \) es la función parte entera de \( x \), entonces para todo \( n\ge m \) se tiene que \( \left|x^{n}-0\right| < \epsilon \) y así \( \{x^n\}\to 0 \) si \( |x|<1 \).

Para ver que \( \{1+x+x^2+\cdots+x^n\} \to \dfrac{1}{1-x} \) observamos que \( \left|1+x+x^2 +\cdots+x^n -  \dfrac{1}{1-x}\right | = \dfrac{|x|^{n+1}}{1-x}  \). Pero, teniendo en cuenta que \( 0<1-|x|\le 1-x  \) y escribiendo \( |x|=\dfrac{1}{1+\rho} \) tenemos que

\[  \left|1+x+x^2 +\cdots+x^n -  \dfrac{1}{1-x}\right | = \dfrac{|x|^{n+1}}{1-x} \le  \frac{|x|}{1-|x|}|x|^n = \frac{1}{p}|x|^n < \frac{1}{np^2} \le \frac{1}{mp^2} < \epsilon  \]

siempre que \( n\ge m \)  y que \( m=\dfrac{1}{\rho^2\epsilon} \) o, mejor aún, si \( m = E\left(\frac{1}{\rho^2 \epsilon}\right)+1 \).


Una manera de obtener la expresión a la que converge es notar que, en general, dado \[ x\in \Bbb R \], para \[ n\in \Bbb N \] tenemos que

\[ (x-1)\sum_{k=0}^n x^k  = \sum_{k=0}^n x^{k+1} - \sum_{k=0}^n x^k = \sum_{k=1}^{n+1} x^k  - \sum_{k=0}^n x^k = x^{n+1} -1    \] y así \[ S_n = \sum_{k=0}^n x^k = \frac{x^{n+1}-1}{x-1}\quad \forall \, x \in \Bbb{R}\setminus \{1\}, \space \forall \, n \in \Bbb N \]. 

Ahora bien, \[ \lim\{S_n\} = \lim_{n\to\infty} \sum_{k=1}^{n} x^k = \lim_{n\to\infty}  \frac{x^{n+1}-1}{x-1}  \] y este límite existe si \[ |x|<1 \], condición que ya habíamos obtenido para que la serie \( \{S_n\} \) fuese convergente. Así, si \( |x|<1 \) entonces \[ \lim\{S_n\} =\frac{1}{1-x} \]
\[  e^{H_n}=\prod_{k=1}^n e^{1/k}\gt\prod_{k=1}^n\left(1+\frac{1}{k}\right)=n+1 \therefore H_n\gt\log(n+1) \]

13 Enero, 2022, 10:44 am
Respuesta #1

Juan Pablo Sancho

  • Moderador Global
  • Mensajes: 6,250
  • País: es
  • Karma: +0/-0
  • Sexo: Masculino
Un camino más corto para \( |x| < 1 \) sería que:
\( s_n = 1+x+x^2 + \cdots + x^n \)
\( x \cdot s_n = x + x^2 + \cdots x^{n+1}  \)
Luego \( s_n - x \cdot s_n = s_n \cdot (1-x) = 1-x^{n+1}  \)
Entonces \( s_n = \dfrac{1}{1-x} - \dfrac{x^{n+1}}{1-x}  \)
Spoiler
Se puede hacer de varias maneras ahora.
Si \( x = 0 \) es fácil.
Si \( x \neq 0  \) entonces \( \displaystyle\lim_{n \to +\infty} \dfrac{|x|^{n+1}}{|x|^n} = |x| < 1 \) luego \(   \displaystyle \lim_{n \to +\infty} |x|^n = 0 \).

También se puede hacer por el Binomio de Newton, que \( |x| = \dfrac{1}{b}  \) con \( b > 1 \) queda:
\( b^n = (1+s)^n = \displaystyle \sum_{k=0}^n {n \choose k} s^k > 1 + n \cdot s  \)
También se puede por la desigualdad de Bernoulli , para \( q > -1  \) tenemos \( (1+q)^n \geq 1+n \cdot q \)
[cerrar]

13 Enero, 2022, 11:11 am
Respuesta #2

Samir M.

  • Physicsguy.
  • $$\Large \color{#5b61b3}\pi\,\pi\,\pi\,\pi\,\pi$$
  • Mensajes: 1,087
  • País: es
  • Karma: +0/-0
  • Sexo: Masculino
  • I'm back.
 :aplauso: :aplauso:

Yo quería dar una solución muy detallada y elemental en el sentido de que no involucrase nada más allá que la definición de convergencia para una sucesión y, por supuesto, la desigualdad de las medias, que a mí me encanta y siempre me es muy útil para encontrar acotaciones. Usar el binomio de Newton mola (de hecho así es como la vi en su día por primera vez) pero me parece muy bestia :D :D
\[  e^{H_n}=\prod_{k=1}^n e^{1/k}\gt\prod_{k=1}^n\left(1+\frac{1}{k}\right)=n+1 \therefore H_n\gt\log(n+1) \]

13 Enero, 2022, 12:14 pm
Respuesta #3

Juan Pablo Sancho

  • Moderador Global
  • Mensajes: 6,250
  • País: es
  • Karma: +0/-0
  • Sexo: Masculino
La otra opción era la desigualdad de Bernoulli , para no usar el Binomio.